1961 AHSME Problems/Problem 24

Revision as of 20:50, 22 May 2018 by Rockmanex3 (talk | contribs) (Solution to Problem 24)
(diff) ← Older revision | Latest revision (diff) | Newer revision → (diff)

Problem 24

Thirty-one books are arranged from left to right in order of increasing prices. The price of each book differs by $\textdollar{2}$ from that of each adjacent book. For the price of the book at the extreme right a customer can buy the middle book and the adjacent one. Then:

$\textbf{(A)}\ \text{The adjacent book referred to is at the left of the middle book}\qquad \\ \textbf{(B)}\ \text{The middle book sells for \textdollar 36} \qquad \\ \textbf{(C)}\ \text{The cheapest book sells for \textdollar4} \qquad \\ \textbf{(D)}\ \text{The most expensive book sells for \textdollar64 } \qquad \\ \textbf{(E)}\ \text{None of these is correct }$

Solution

Let the leftmost book cost $c$ dollars. The 16th book (the middle book) costs $c+30$ dollars, and the 31st book costs $c+60$ dollars.

If the book to the left of the middle book is used, the equation to solve for the price of the leftmost book is $c+30+c+29=c+60$. Solving yields $c=1$, so the price of the middle book is $\textdollar{31}$ and the price of the rightmost book is $\textdollar{61}$.

If the book to the right of the middle book is used, the equation to solve for the price of the leftmost book is $c+30+c+31=c+60$. Solving yields $c=-1$, which can not happen.

Thus, the answer is $\boxed{\textbf{(A)}}$.

See Also

1961 AHSC (ProblemsAnswer KeyResources)
Preceded by
Problem 23
Followed by
Problem 25
1 2 3 4 5 6 7 8 9 10 11 12 13 14 15 16 17 18 19 20 21 22 23 24 25 26 27 28 29 30 31 32 33 34 35 36 37 38 39 40
All AHSME Problems and Solutions

The problems on this page are copyrighted by the Mathematical Association of America's American Mathematics Competitions. AMC logo.png